Lange Emergency Medicine

¡Supera tus tareas y exámenes ahora con Quizwiz!

Which of the following best defines the peripheral wedge-shaped consolidation on the pleural surface observed in a patient with a pulmonary embolism? (A) Hampton hump (B) pleural effusion (C) atelectatic lesion (D) Westermark sign (E) reticular pattern

(A) A Hampton hump generally represents a focal area of hemorrhage within the lung or an actual pulmonary infarction. It is a wedge-shaped, dense, consolidated area on the pleural surface of the chest wall. A Westermark sign is a regional area of decreased pulmonary vascularity. Other more common findings of pulmonary embolism on a chest radiograph include atelectasis, elevated hemidiaphragm, patchy consolidation, and pleural effusions

A 34-year-old male patient presents to the ED following an episode of "rough" sex with his girlfriend. He complains of a painful swollen penis. Which of the following descriptions best defines a penile rupture? (A) rupture of the corpus cavernosum when the tunica albuginea is torn (B) rupture of the dorsal penile artery into the tunica albuginea (C) rupture of the corpus spongiosum from a tear in the dorsal penile vein (D) rupture of the tunica albuginea from a tear into the corpus cavernosum (E) rupture of the corpus cavernosum when the corpus spongiosum is torn

(A) A penile rupture is a traumatic rupture of the corpus cavernosum when the tunica albuginea is torn. During vigorous sexual intercourse, a patient commonly will hear a snapping sound followed by localized pain, detumescence, and slowly progressive penile hematoma.

A 64-year-old man presents to the ED with decreased visual acuity, red eye, and a "steamy" or hazy cornea. What is the most likely diagnosis? (A) acute narrow-angle glaucoma (B) iritis/uveitis (C) orbital cellulitis (D) allergic conjunctivitis (E) episcleritis

(A) Acute narrow-sangle glaucoma should be recognized as an ophthalmic emergency. It is characterized by a sudden onset of severe pain localized to the affected eye. Common associated visual symptoms include halos around lights, blurriness, and scotomas. Other associated symptoms include nausea and 658 vomiting. The typical physical examination findings reveal a red eye with fixed, mid-dilated pupil, corneal clouding, and a shallow anterior chamber. Anterior uveitis is inflammation of the anterior segment of the eye. Anterior uveitis includes iritis (inflammation that involves only the iris) and iridocyclitis (inflammation of both the iris and ciliary body). Physical examination findings include ciliary flush (ie, circumcorneal perilimbal injection of the episcleritis and scleral vessels) conjunctival injection, and cells may be present in the anterior chamber. The pupil on the affected side is often small and irregular. Direct and consensual light reflex will cause pain on the affected side to increase. Orbital cellulitis is recognized as a soft tissue infection that extends deep into the fascia and eye orbit. Clinical findings include ocular pain, limitation of eye movement, lid edema, and proptosis, tenderness of the globe, decreased visual acuity, increased ocular pressure, and pupillary paralysis. Allergic conjunctivitis is most common signs and symptoms include red or injected conjunctiva, chemosis, eye drainage, and pruritus. It is mediated by a hypersensitivity exposure. Episcleritis is the inflammation of the connective tissue between the sclera and the conjunctiva. Episcleritis is commonly described as an irritation rather than a true pain. In addition, the orbital vessels blanch with topical neosynephrine.

A 40-year-old man presents to the ED with acute blunt chest and abdominal trauma following a motor vehicle crash. The patient presented with jugular venous distention, decreased BP, and muffled heart tones. Which of the following is the most likely diagnosis? (A) pericardial tamponade (B) tension pneumothorax (C) myocardial rupture (D) aortic rupture (E) myocardial contusion

(A) Any patient who has sustained a penetrating wound or blunt trauma to the thorax or upper abdomen should be suspected of having a diagnosis of pericardial tamponade. The most common signs of pericardial tamponade are hypotension and tachycardia-associated elevation in central venous pressure. Beck triad of pericardial tamponade consists of hypotension, distended neck veins, and distant heart sounds. A tension pneumothorax is an accumulation of air under pressure within the pleural cavity. The air under pressure shifts the 660 mediastinum to the opposite hemithorax and compresses the contralateral lung and great vessels. A myocardial rupture refers to an acute traumatic perforation of the ventricles and atria. Acute myocardial rupture also includes rupture of the interventricular septum, pericardium, chordae, interatrial septum, and papillary muscles and valves. The most common vessel injured in an acute blunt trauma is the thoracic aorta. Deceleration injuries most commonly injure the thoracic aorta because the descending aorta is relatively fixed by the attachments of the intercostal arteries and ligamentous arteriosum. Myocardial contusion will usually demonstrate direct areas of hemorrhage in the anterior wall of the right ventricle and atria

Which of the following interventions is the most important factor in surviving an out-of-hospital cardiac arrest? (A) immediate airway control with intubation (B) early defibrillation (C) aggressive management of hypotension (D) epinephrine usage (E) rapid transport to appropriate facility

(B) Survival of prehospital cardiac arrest is most notably improved by defibrillation and good cardiopulmonary resuscitation (CPR). Emergent airway management, the use of epinephrine, prompt treatment of hypotension, and rapid transport to the most appropriate facility only moderately increase survival rates when compared with early defibrillation or CPR.

A 2-year-old child was brought to the ED after swallowing a button battery from a watch. Which of the following statements is true regarding button battery ingestion? (A) A button battery lodged in the esophagus is a true emergency because of the extremely rapid action of the alkaline substance on the mucosa. (B) Button battery ingestion is essentially a benign ingestion because of the unlikelihood of the battery dissolving. (C) Button battery ingestion is a minor emergency that can often be treated with a Foley balloon technique extraction. (D) Most button batteries, even if symptomatic, can be left to pass through the GI tract naturally by peristalsis. (E) Surgical removal of the button battery is always indicated, even if the patient is asymptomatic.

(A) Button battery ingestion may cause significant complications in as little as 4 to 6 hours due to the rapid action of alkaline in the battery. Severe burns of the esophagus or perforation may occur. A plain radiograph of the abdomen should be obtained first to localize the battery. A battery lodged in the esophagus should be removed emergently with endoscopy. A surgical consult may be indicated for symptomatic ingestions past the esophagus.

A 45-year-old woman presents to the ED with *acute* painless loss of vision, photophobia associated with a smaller unilateral pupil on the involved side. Which of the following is the most likely diagnosis? (A) central retinal artery occlusion (B) central retinal vein occlusion (CRVO) (C) iritis/uveitis (D) retrobulbar hemorrhage or hematoma (E) hyphema

(A) Central retinal artery occlusion is characterized by *acute* visual loss usually attributed to ischemic or thrombus to the major retinal arterial blood supply. Typically, the patient presents with *sudden*, painless onset of markedly decreased unilateral loss of vision. Physical examination findings include significant decrease in visual acuity, relative afferent pupillary defect (ie, Marcus Gunn pupil), and a pale retina with a red spot that is visible on funduscopic examination. Central retinal vein occlusion (CRVO) is characterized by painless, unilateral vision loss of varying severity, *slower* onset of decreased vision than with arterial occlusion, retinal hemorrhages, cotton wool spots, and macular edema. Physical examination findings include ciliary flush (ie, circumcorneal perilimbal injection of the episcleritis and scleral vessels) conjunctival injection and cells may be present in the anterior chamber. The pupil on the affected side is often small and irregular. Direct and consensual light reflex will cause pain on the affected side to increase. Retrobulbar hemorrhage is associated with decreased ocular range of motion, decreased vision, ptosis of the lid, and increased pressure in the globe raising intraocular pressure. The high pressure decreases retinal artery perfusion, which results in retinal ischemia. The patient presents with decreased visual acuity, proptosis, and a dilated nonreactive pupil. A hyphema is caused by bleeding from the vasculature of the iris usually precipitated by trauma. Blood is often visualized in the anterior chamber and can be seen via slit lamp evaluation. Symptoms usually consist of pain, photophobia, and decreased vision. Intraocular pressures may increase as well. The major clinical consideration is the potential of reoccurring bleeding.

A patient presents with an acute asthmatic attack. Multiple doses of inhaled adrenergic agents are used but the patient continues to have bronchospasms. Which of the following medications is most paramount in the treatment of bronchospasms? (A) corticosteroids (B) anticholinergics (C) theophylline (D) magnesium (E) leukotriene modifiers

(A) Corticosteroids remain one of the keystones of treatment for asthma. Steroids are thought to decrease airway inflammation and restore βadrenergic responsiveness. The peak onset of inflammatory effects is delayed at least 4 to 8 hours following oral or intravenous administration. Theophylline is no longer considered a first-line therapy for acute asthma because of its high risk for toxicity, especially when combined with β-adrenergic drugs. Magnesium does have some bronchodilating effects and can be used in the management of acute asthma. Magnesium should be used only after standard therapy has been unsuccessful. Leukotriene modifiers decrease inflammation, edema, mucous secretion, and bronchoconstriction, thereby diminishing the need for short- acting β2 agonists; however, their role in the ED setting has not proven to be of significant benefit in the setting of acute bronchospasms

Which of the following therapeutic interventions is the most beneficial in lowering increased intracranial pressure (ICP) associated with a subarachnoid hemorrhage? (A) nimodipine (B) mannitol (C) hyperventilation (D) corticosteroids (E) phenobarbital

(A) In treating subarachnoid hemorrhage (SAH), the use of nimodipine has established parameters of reducing disability and death by 55% as compared with the placebo. Mannitol is a good therapeutic agent for reducing ICP, however, not specific to SAH. Although hyperventilation is noted to reduce ICP, it is no longer recommended as a preventative measure in the initial period after a traumatic brain injury (TBI). Steroids have no clinical indications in the treatment of TBI or increased ICP. Use of a barbiturate coma is not indicated in the ED setting, although there is evidence of its benefit in stable patients who have failed to respond to other ICP-lowering treatment modalities.

A 68-year-old patient presents to the ED with palpitations and dizziness. The ECG demonstrates three or more differently shaped P waves; varying PP, PR, and RR intervals; and atrial rhythm usually between 100 and 180. What is the most likely dysrhythmia? (A) MAT (B) AF (C) ventricular tachycardia (VT) (D) sinus dysrhythmia (E) supraventricular tachycardia

(A) MAT is associated with the following ECG characteristics: (1) three or more differently shaped P waves; (2) varying PP, PR, and RR intervals; and (3) atrial rhythm usually between 100 and 180. AF is associated with the following ECG characteristics: (1) fibrillatory waves of atrial activity, best seen in leads V1, V2, V3, and aVF and (2) irregular ventricular response, usually around 170 to 180 in patients with a healthy AV node. Ventricular tachycardia is associated with the following ECG characteristics: (1) wide QRS complexes; (2) rate greater than 100; (3) usually regular rhythm; and (4) a constant QRS axis. A sinus dysrhythmia is associated with the following ECG characteristics: (1) normal sinus P waves and PR intervals, (2) 1:1 AV conduction, and (3) variation of at least 0.12 second between the shortest and longest PP interval. Supraventricular tachycardia usually occurs at a rate of 100 to 250 bpm with a regular rhythm. (Bolton, 2004, pp. 181-191)

Spontaneous esophageal rupture following forceful vomiting after overindulging in food and alcohol is known as: (A) bezoar (B) Boerhaave syndrome (C) Burger sign (D) Brudzinski sign

(B) Boerhaave syndrome, postemetic rupture, and spontaneous esophageal rupture are synonymous terms. The most common site of injury is the distal esophagus, which demonstrates a longitudinal tear occurring in the left posterolateral aspect. Most cases occur in middle-aged men after they have overindulged in food and alcohol. Burger sign is defined as a physical examination finding of advanced peripheral vascular disease. Brudzinksi sign is a physical examination diagnostic maneuver in which hip flexion occurs with passive flexion of the neck and is interpreted as a positive meningeal sign. Bezoars (undigested vegetable matter) represent an intraluminal obstruction in those having undergone prior surgeries such as pyloric resection

A 65-year-old man with chronic obstructive pulmonary disease (COPD) taking chronic theophylline therapy presents to the ED with palpitations, chest pain, and the feeling that his heart is beating irregularly after starting erythromycin for bronchitis. What is the likely dysrhythmia? (A) multifocal atrial tachycardia (MAT) (B) atrial fibrillation (AF) (C) atrial flutter (D) sinus bradycardia (E) Mobitz type II heart block

(A) MAT is defined as a chaotic, irregular rhythm with atrial rates of 100 to 150 bpm. Typically, there are more than two foci of impulse formation with at least three distinctly different P waves with varying P" R, RR, and P' P' intervals. MAT is commonly associated with COPD, theophylline toxicity, and β-adrenergic agonist therapy. AF is a totally chaotic atrial rhythm with multiple microreentry circuits of atrial rates from 300 to 600 impulses/min. The most common causes of AF include ischemic heart disease, valvular heart disease, pericarditis, and hyperthyroidism. Atrial flutter is characterized by regular atrial depolarization rates of 250 to 350 bpm, with varying degrees of atrioventricular block. Common causes of atrial flutter include atherosclerotic heart disease, myocardial infarction, thyrotoxicosis, pulmonary embolism, mitral valve disease, congestive heart failure, and metabolic derangements. Sinus bradycardia is a regular rhythm with atrial and ventricular rates of less than 60 bpm with normal P-wave morphology and PR duration. Sinus bradycardia can be found in healthy adults or it may be associated with pathologic conditions such as hypothermia, excessive parasympathetic tone, carotid sensitivity, or myocardial infarction. A type II second-degree AV block or Mobitz II block is characterized by a sudden interruption of AV conduction without prior prolongation of the PR interval. Mobitz II is often associated with a variety of acute and chronic diseases such as anterior wall ischemia. (Yearly and Delbridge, 2006, pp. 1228-1229)

The drug of choice for treating hypertensive encephalopathy in the nonpregnant patient is (A) sodium nitroprusside (B) labetalol (C) esmolol (D) IV nitroglycerin (E) hydralazine

(A) Most of the medications listed are a good option for hypertensive emergencies. Sodium nitroprusside is the most widely used/available, is a rapidly acting arterial and venous dilator, and is the drug of choice for most hypertensive emergences unless there is severe kidney disease. Labetalol is an excellent drug for hypertensive emergencies. It is a competitive, selective alpha - 1-blocker and a competitive, nonselective β-blocker, with the β- blocking action four to eight times that of alpha blocking. Esmolol is an ultrashort-acting β1 selective adrenergic blocker with rapid distribution and elimination. Nitroglycerin causes both arterial and venous dilation, with a greater effect on the venous system. The onset of action with nitroglycerin is almost immediate when given IV, and the half-life is 4 minutes. Hydralazine is a direct arterial dilator, with the onset of action within 10 minutes when given IV and duration of action 4 to 6 hours.

A patient presents to the ED with a dislocated shoulder. Nitrous oxide is the drug selected for sedation and analgesia during reduction. Which of the following is true with regard to the administration of nitrous oxide for short- term painful procedures in the ED? (A) A 50:50 concentration of nitrous and oxygen should be used. (B) Never administer oxygen with nitrous oxide. (C) Nitrous oxide concentrations should always be less than 30%. (D) Higher altitudes require lower concentrations of nitrous oxide. (E) Nitrous oxide is not approved for ED use.

(A) Nitrous oxide may be used for both sedation and analgesia in the emergency department, as long as it is mixed with at least 30% oxygen to prevent hypoxia. Therapeutic concentrations of nitrous oxide include those in the 30% to 50% range (maximum 70%). Concentrations lower than 30% may not be effective in this setting. Higher concentrations of nitrous oxide are required at higher altitudes. Younger children (younger than 8 years) may not gain a therapeutic effect from nitrous oxide

A 59-year-old cancer patient presents to the ED with fever, pneumonia, hypotension, and tachycardia. Investigative studies include hyper-kalemia, hyponatremia, and hypoglycemia. What is the most likely concomitant diagnosis in this scenario? (A) adrenal insufficiency (B) syndrome of inappropriate antidiuretic hormone (SIADH) (C) Cushing syndrome (D) hypothyroidism (E) hyperparathyroidism

(A) Primary adrenocortical insufficiency (Addison disease) is characterized by inadequate secretion of cortisol, aldosterone, or both most frequently resulting from autoimmune-induced destruction of the adrenal glands. Other causes of Addison disease include tuberculosis, viral infections, 647 carcinomatous destruction of the adrenals, adrenal infarction from arteritis or thrombosis, and adrenal hemorrhage. Laboratory results characteristically include hyperkalemia, hyponatremia, hypochloremia, hypoglycemia, elevated BUN/creatinine ratio (prerenal azotemia), anemia, decreased 24-hour urinary cortisol, 17-hydroxycorticosteroid, and 17-ketosteroids and increased ACTH (adrenocorticotropic hormone; primary adrenocortical insufficiency). SIADH is a syndrome of antidiuretic hormone (ADH) excess, which causes water retention and sodium loss. Common causes of SIADH include malignant tumors, intracranial hemorrhage, hydrocephalus, meningitis, brain abscess, chlorpropamide, thiazide diuretics, desmopressin, and chemotherapeutic agents. Laboratory values of SIADH include hyponatremia, urinary osmolarity greater than serum osmolarity, and urinary sodium level usually higher than 30 mEq/L. Cushing syndrome is characterized by glucocorticoid excess secondary to exaggerated adrenal cortisol production or chronic glucocorticoid therapy. Causes of primary glucocorticoid excess include idiopathic, tuberculosis, fungal infections, adrenal hemorrhage, congenital adrenal hyperplasia, sarcoidosis, amyloidosis, HIV/AIDS, and metastatic diseases. Common laboratory results include hypokalemia, hypochloremia, metabolic alkalosis, hyperglycemia, and hypercholesterolemia. Primary hypothyroidism (ie, thyroid gland dysfunction) causes most cases of hypothyroidism. Secondary hypothyroidism includes pituitary dysfunction, postpartum necrosis, neoplasm, and infiltrative diseases causing a deficiency of thyroid-stimulating hormone (TSH or thyrotropin). Tertiary causes of hypothyroidism include hypothalamic diseases such as granuloma, neoplasm, or irradiation causing deficiency of thyroid releasing hormone. Common laboratory results of hypothyroidism are increased TSH, hyponatremia, increased cholesterol triglycerides, and liver function tests. Primary hyperparathyroidism is the result of oversecretion of PTH, which in turn causes hypercalcemia. A parathyroid adenoma is the primary etiology of hyperparathyroidism. Laboratory findings include hypercalcemia, hypophosphatemia, hyperchloremia, elevated serum alkaline phosphatase level, and hypercalcuria

A 64-year-old woman presents to the ED with dyspnea and exertional fatigue. There is a high clinical suspicion for pulmonary embolism. The ventilation/perfusion ( ) lung scan was read as "low probability" for a pulmonary embolism. Of the following, which would be the most appropriate next step? (A) Obtain a pulmonary angiogram if the lower extremity ultrasound is negative for deep vein thrombosis (DVT). (B) Initiate intravenous (IV) heparin without further testing. (C) Send the patient home with instructions to follow up with her physician the next day for further outpatient work-up. (D) Obtain an echocardiogram to assess right atrial pressures. (E) Repeat the lung scan.

(A) Pulmonary thromboembolism (PTE) is primarily the result of clot migration from DVT. Even with current technology, the diagnosis of DVT and PTE is rather evasive and difficult. Ventilation/perfusion scanning (V/Q) is relatively nondiagnostic in most cases of PTE. Only 41% of patients with PTE confirmed through pulmonary angiography will have a high-probability lung scan. In general, a patient has a less than 5% chance of having a PTE if the scan is read as "normal." A "high-probability" scan has a more than 85% chance that a PTE is present. A scan interpreted as "low probability" or "nondiagnostic" will have a 15% to 85% chance of having a PTE. Therefore, a nondiagnostic or low-probability scan is useless in ruling in or ruling out a PTE. Computed tomography offers a relatively good way of diagnosing PTE; however, the sensitivity ranges from 40% to 65% and the negative predictive value is around 82% when compared with pulmonary angiography. The pulmonary angiogram is considered the last resort for diagnosing a PTE. A "positive" angiogram provides essentially 100% certainty of the diagnosis of PTE. A "negative" angiogram provides more than 90% certainty that PTE is excluded. However, technical and patient factors such as dye concerns and movement can significantly alter the diagnostic ability of angiogram. (Kline and Runyon, 2006, pp. 1371-1381)

A patient presents to the ED after being bitten by an unknown "insect" while camping. The pain began as a pinprick sensation at the bite site and spread quickly to include the entire bitten extremity. The bite wound became erythematous 45 minutes after the bite. The bite evolved into a target lesion and the patient complains of muscle cramp-like spasms in the large muscle groups. Which of the following is the most likely cause? (A) black widow spider (B) hobo spider (C) brown recluse spider (D) tarantula (E) scorpion

(A) The black widow spider (Latrodectus) is found in many areas of the United States. Its bite produces immediate pain and pinprick sensations that soon encompass the entire extremity. Erythema of the bitten area develops usually within 1 hour and in about half of the cases quickly evolves into a target pattern. Patients frequently complain of cramp-like spasms in the large muscle groups. The physical examination rarely exhibits muscle rigidity, and serum creatine kinase concentrations usually are not elevated significantly. The brown recluse (Loxosceles) spider bites are difficult to identify. The bite lesion is usually mildly erythematous and may become firm and heal with little scarring over several days to weeks. Occasionally, the lesion may become necrotic over 3 to 4 days with subsequent eschar formation. The hobo spider (Tegenaria) usually causes a painless local reaction similar to that of the brown recluse spider. Blisters eventually develop that rupture, leaving an encrusted cratered wound. A tarantula bite typically causes pain and local swelling at the site. Treatment consists of local wound care. Scorpions 665 (Scorpionida) present with a multitude of local and systemic manifestations. Some of these manifestations include pain, paresthesia, cranial nerve and somatic motor dysfunction, uncontrolled jerking, restlessness, pharyngeal incoordination, and respiratory compromise.

A child falls on an outstretched hand. She complains of pain and swelling to the wrist. The radiograph demonstrates a buckling of the cortex to the distal radius. What is your diagnosis? (A) torus fracture (B) greenstick fracture (C) complete fracture (D) plastic deformation

(A) The developing bones of the child are more pliable and flexible than an adult mature bone. In a torus fracture, there is a buckling of the cortex of the bone without complete disruption of the cortical segment. Multiple radiographic views may be necessary to make the diagnosis in small, nondisplaced fractures.

Which of the following is the most common cause of nontraumatic cardiac tamponade? (A) metastatic malignancy (B) uremia (C) acute idiopathic pericarditis (D) hemorrhage (anticoagulant use) (E) bacterial or tubercular pericarditis

(A) The most common cause of nontraumatic cardiac tamponade is metastatic malignancy. Common symptoms include dyspnea and profound exercise intolerance. Physical examination findings include tachycardia, low systolic arterial BP with a narrow pulse pressure, and pulsus paradoxus. Less common causes of nontraumatic tamponade include acute or chronic idiopathic pericarditis, uremia, bacterial or tubercular pericarditis, hemorrhage (from anticoagulant use), systemic lupus erythematosus, radiation treatments, and 648 myxedema.

A 1-month-old infant presents to the ED with lethargy, tachycardia, fever, rash, and leukocytosis. The diagnosis of bacterial meningitis is suspected and a lumbar puncture was performed. What organisms should the antibiotic cover in this case? (A) Listeria monocytogenes, group B streptococcus, and Escherichia coli 624 (B) E coli, Klebsiella, and Pseudomonas (C) group B streptococcus, enterococcus, and chlamydia (D) Listeria, Staphylococcus, and Campylobacter species (E) group A streptrococcus and Salmonella species

(A) The most common causes of bacterial meningitis in the infants aged 0 to 4 weeks include group B streptococci, E coli, and L monocytogenes. Pathogens commonly infecting infants 4 to 12 weeks include group B streptococci, E coli, L monocytogenes, H influenzae, and S pneumoniae. Pathogens infecting children and young adults 3 months through 17 years include S pneumoniae and N meningitidis. With the introduction of the conjugate vaccine for H influenzae, dramatic reductions in incidences of H influenzae meningitis has been seen. Finally, likely pathogens in adults 18 years and older include S pneumoniae and N meningitides and less frequently L monocytogenes.

A 44-year-old AIDS patient being treated for Pneumocystis jiroveci pneumonia presents to the ED with an acute onset of confusion, pallor, diaphoresis, and tachycardia. What is the most likely cause of the patient's symptoms? (A) hypoglycemia (B) meningitis (C) subarachnoid hemorrhage (SAH) (D) hypoxemia (E) mucous plugging

(A) The symptoms described are due to hypoglycemia caused by pentamidine isethionate. Pentamidine is a common treatment for pneumocystis in the AIDS patient. Pentamidine is an antiprotozoal agent that inhibits synthesis of DNA, RNA, phospholipids, and proteins. Common side effects include hypoglycemia, renal impairment, leukopenia, hepatotoxicity, nausea, anorexia, 645 hypotension, fever, and rash. Monitor metabolic parameters such as BUN level, creatinine level, glucose level, CBC, platelet count, liver function tests, and calcium level regularly while on pentamidine therapy. (Rothman et al., 2006, p. 2079)

Cardiac enzymes in a patient with a 2-hour history of chest pain secondary to an acute myocardial infarction would commonly demonstrate which of the following findings? (A) normal creatine kinase (CK-MB) and troponin I levels (B) elevated troponin I and normal CK-MB levels (C) elevated CK-MB and normal troponin I levels (D) normal myoglobin with elevated CKMB levels (E) elevated myoglobin, troponin I, and CK-MB levels

(A) Two hours after the onset of an acute myocardial infarction, the cardiac enzymes would most commonly demonstrate a normal troponin and CK-MB levels. Cardiac troponin I and CK-MB levels elevate in 3 to 12 hours after the onset of myocardial infarction. Serum myoglobin level elevates 1 to 4 hours after the onset of myocardial infarction.

A 4-week-old infant presents with several hours of persistent crying. A markedly red and edematous left third toe is found upon examination. What anatomical location of the affected digit would be the most appropriate site to perform an emergent surgical release of the strangulation? (A) anterior (B) dorsal (C) lateral (D) proximal (E) superficial

(B) An infrequent injury of the digits seen in infancy is known as the hair tourniquets syndrome, in which a strand of hair inadvertently gets wrapped around a digit (usually toes). Local tissues of the affected digit become strangulated and ischemia subsequently develops. To resolve vascular compromise, the hair must be completely excised to restore perfusion to the affected digit. A midline longitudinal incision on the extensor surface (dorsal) of the digit deep enough to cut the extensor ligament is considered to customary approach to accomplish reperfusion. Complete removal of any remaining hair strands should be attempted with the use of forceps without teeth.

Which of the following is the most common cause of erosive esophagitis? (A) radiation therapy (B) gastrointestinal reflux (C) candida (D) herpes simplex virus (E) pills

(B) Gastroesophageal reflux disease (GERD) has been established as the most widespread etiology of inflammation of the esophagus, also known as esophagitis. Additional commonly occurring sources of esophagitis comprise pill esophagitis, esophageal damage and inflammation associated with the effects of alkaline or acidic ingestions, radiation, and infectious agents. It is suspected that the true incidence of pill esophagitis is underreported. Candida is one of the primary infectious pathogens for the infection of the esophagus. As more individuals in immunocompromised immune states receive preventative therapy for opportunistic fungal infections, viral esophagitis has increased in prevalence. Herpes simplex I (HSV) and cytomegalovirus (CMV) have been established as the most common viral agents.

A 58-year-old man with multiple myeloma presents to the ED with altered mental status, hypertension, back pain, and constipation. These findings are suggestive of which of the following medical conditions? (A) hyperkalemia (B) hypercalcemia (C) hypomagnesemia (D) hypoglycemia (E) hyponatremia

(B) Hypercalcemia associated with malignancies is commonly due to increased bone resorption through osteoclastic factors, parathyroid hormone (PTH) factors, prostaglandins, peptides, steroids, and direct erosion by tumor cells. Common neoplasms include multiple myeloma, lymphosarcoma, adult T- cell lymphoma, and Burkitt lymphoma. Symptoms of hypercalcemia are variable depending on the degree elevation. Typical symptoms include constipation, anorexia, vomiting, confusion, obtundation, psychosis, nephrolithiasis, renal insufficiency, myopathy, back pain, weakness, and hypertension. Hyperkalemia may produce manifestations of weakness, irritability, paresthesias, paralysis, cardiac arrhythmia, and decreased deep tendon reflexes. Hypomagnesemia symptoms include weakness, fasciculations, tremors, convulsions, delirium, coma, hyperreflexia, and cardiac arrhythmias. Hypoglycemia commonly presents with varying degrees of diaphoresis, anxiety, tremors, tachycardia, palpitations, fatigue, syncope, headache, visual disturbances, hemiplegia, and seizures. Hyponatremia may present as confusion, muscle cramps, anorexia, nausea, lethargy, seizures, and coma depending on the degree and rapidity of onset. (Gibbs and Taval, 2006, pp. 1944-1947)

A 42-year-old woman was brought to the ED from a psychiatric facility for an evaluation following a brief "seizure." The psychiatric staff reports that she has been confused and complaining of thirst for the past 5 days. What is the most likely diagnosis? (A) idiopathic hypoglycemia (B) psychogenic polydipsia (C) brain tumor (D) new-onset epilepsy (E) psychogenic cerebritis

(B) In this scenario, the patient's sodium level dropped severely and rapidly secondary to excessive water intake. In this setting, patients may present with confusion, muscle cramps, lethargy, anorexia, nausea, seizures, and coma. Other causes of euvolemic hyponatremia include syndrome of inappropriate secretion of antidiuretic hormone (SIADH), renal failure, glucocorticoid deficiency (hypopituitarism), hypothyroidism, and multiple medications such as thiazide diuretics. (Gibbs and Taval, 2006, p. 1934)

Le Fort I facial fracture is best described as (A) a fracture involving the maxilla, the nasal bones, and the medial aspects of the orbits (B) a fracture involving the maxilla at the level of the nasal fossa (C) a fracture involving the maxilla, zygoma, nasal bones, ethmoids, vomer, and all lesser bones of the cranial base (D) A fracture involving frontal bones and bones of the midface

(B) Le Fort I fractures involve the maxilla at the level of the nasal bones. A Le Fort II fracture involves several facial bones, including the maxilla, the nasal bones, and the medial aspects of the orbits. A Le Fort III fracture includes aspects of the maxilla, zygoma, nasal bones, ethmoids, vomer, and lesser bones of the cranial base. It can also be described as a craniofacial dysfunction.

Which of the following diagnostic studies is the most reliable marker of rhabdomyolysis? (A) urine myoglobin (B) creatinine kinase (C) creatinine kinase-MB (D) lactate dehydrogenase (LDH) (E) aldolase

(B) Measuring serum levels of the enzyme creatine phosphokinase (CK or CPK) is the most sensitive marker for evaluating muscle damage as associated with rhabdomyolysis. In rhabdomyolysis, the isoenzyme CK-MB would not be more than 5% of the total CK or CPK. Myoglobin is also associated with muscle injury; however, it is a less sensitive marker than CPK. LDH and aldolase are additional laboratory tests that can be used in evaluating rhabdomyolysis. However, both are less specific than CPK.

A 65-year-old ill-appearing woman presents to the ED with tachycardia, tachypnea, and an arterial pH of 7.10. What is the most likely cause of her high anion gap metabolic acidosis? (A) diabetic ketoacidosis (DKA) (B) lactic acidosis (C) alcoholic ketoacidosis (D) nonketotic hyperosmolar acidosis (E) aspirin poisoning

(B) Metabolic acidosis is characterized by increased production of acids, decreased excretion of acids, or loss of bicarbonate. The etiology of metabolic acidosis is divided into those with a normal anion gap and those associated with an increased anion gap. Metabolic acidosis with an increased anion gap includes lactic acidosis, ketoacidosis, and renal failure. Lactic acidosis, the most common cause of nonhospitalized, nondiabetic metabolic acidosis is due to decreased oxygen delivery to tissues and associated anaerobic metabolism, which results in an increased production of lactate. This lactate production accompanies severe metabolic acidosis. Low tissue perfusion, characteristic of lactic acidosis, may include shock and sepsis. Metabolic acidosis with a normal anion gap is known as hyperchloremic metabolic acidosis. The primary causes of normal anion gap acidosis include renal loss of bicarbonate through proximal tubular acidosis, distal tubular acidosis, hyperkalemic renal tubular acidosis, renal insufficiency, or carbonic anhydrase inhibition and gastrointestinal loss of alkali through diarrhea, pancreatic fistulas, or ureterosigmoidostomy.

Which of the following demographic groups below would be most likely to suffer from a primary tuberculosis (TB) infection presenting as extrapulmonary tuberculosis meningitis? (A) elderly (B) children younger than 2 years (C) young adults (D) alcoholics (E) HIV patients with CD4 counts higher than 350/μL

(B) Six percent of the cases of extrapulmonary TB affect the CNS. The peak incidence of extrapulmonary CNS TB are seen in the pediatric age range of birth to 4 years. Among children younger than 4 years, about 25% will develop extrapulmonary TB manifestations. In recent years, military TB (acute disseminated TB) has become more common in the elderly and those with HIV infections; previously this form of TB was more common in children. The multi-system or miliary form of TB is also seen in chronic alcoholics and those with cirrhotic liver disease.

A patient presents to the ED after being trapped in a house fire. The patient suffered partial thickness burns over the entire anterior chest and abdomen, entire right arm, and the entire right leg. Using the rule of nines, what is the estimated percentage of burn? (A) 36% (B) 45% (C) 48% (D) 54% (E) 72%

(B) The answer is 44% burn. The rule of nines to estimate percentage of burns is as follows: head 9%, anterior trunk 18%, posterior trunk 18%, each leg 18%, each arm 9%, and perineum 1%. CHEST AND Abdomen= 18 Right arm=9 right leg=18

Which of the following medications is responsible for the most drug-related deaths? (A) benzodiazepines (B) tricyclic antidepressants (TCAs) (C) stimulants (D) monoamine oxidase inhibitors (E) lithium

(B) The class of prescription medications responsible for the most drug- related deaths is TCAs. The clinical toxicity is due to the complex pharmacologic activity, low therapeutic index, and general availability. The clinical toxicity is quite variable, ranging from mild antimuscarinic activity to severe cardiotoxicity. Benzodiazepine related overdoses account for few deaths; however, in combination with other agents, they account for significant deaths and disability due to additive effects. Although not reflected in death rates, monoamine oxidase inhibitors have greater toxicity than the newer antidepressants. The toxic effects of lithium are frequently related to drug interactions. Stimulants are associated with significant side effects

A pregnant woman, at 25 weeks' gestation, is involved in a motor vehicle accident. She suffers blunt abdominal trauma. The vital signs are a pulse of 100/min, a respiratory rate of 40/min, and a BP of 78/40. Which of the following is a true statement regarding the management of trauma in pregnancy? (A) Intubation should be avoided in the mother because of barotrauma complications. (B) The first resuscitative efforts should be directed toward the mother. (C) The fetus is less susceptible to hypoxia because of uterine reserve. (D) A stat C-section should be considered in every patient. (E) Fetal distress is not easily recognized.

(B) The first priority is resuscitation of the mother. A secure airway is very important because aspiration is common. Oxygen therapy is critical because of the reduced oxygen reserve and increased oxygen consumption in the mother. A trauma patient can quickly become hypoxic, making the fetus very vulnerable to any reduction in oxygen delivery.

Which of the following maneuvers would be the most helpful in diagnosing an Achilles tendon rupture? (A) pivot shift (B) Thompson-Doherty test (C) Lachman test (D) anterior drawer test (E) posterior drawer test

(B) The most beneficial maneuver in evaluating a patient with an Achilles 656 tendon injury is the Thompson-Doherty test or the Thompson test. This maneuver is performed by squeezing the patient's calf of the affected lower extremity while the patient is lying in a prone position. An intact Achilles is noted by visualizing plantar flexion of the foot while applying the preceding described maneuver. The Lachman test is the more sensitive and specific for establishing anterior cruciate ligament (ACL) injuries. Anterior drawer test and the pivot shift test are also used to evaluate ACL injuries. The posterior drawer test is used to evaluate injuries to the posterior cruciate ligament.

A 40-year-old man slips on the ice, injuring his left arm. He complains of pain and swelling to the midshaft humeral region. The physical examination reveals a wrist drop on the injured side. Which nerve is most likely injured? (A) ulnar (B) radial (C) median (D) axillary (E) subclavian

(B) The most common nerve injured with a humeral shaft fracture is the radial nerve. The radial nerve runs in close proximity to the posterior midhumeral shaft. A radial nerve injury is evident by a wrist drop

A 48-year-old patient presents to the ED with tachycardia, palpitations, and 623 restlessness. The diagnosis of thyroid storm is confirmed and general supportive care is provided. Which of the following is the correct sequence of medications to treat thyroid storm? (A) propylthiouracil (PTU)—propranolol—iodide (B) propranolol—PTU—iodide (C) propranolol—iodide—PTU (D) iodide—propranolol—PTU (E) iodide—PTU—propranolol

(B) Thyroid storm is characterized by an abrupt, severe, exacerbation of hyperthyroidism. Hyperthyroidism (ie, thyrotoxicosis, thyrotoxic crisis, and thyroid storm) refers to varying degrees of thyroid hyperfunction. Common causes of thyroid storm include major stress (eg, infection, surgery, DKA, myocardial infarction) in a patient with undiagnosed hyperthyroidism and inadequate therapy in a hyperthyroid patient. Patients typically present with fever, anxiety, agitation, psychosis, hyperhidrosis, heat intolerance, weakness, muscle wasting, palpitations, diarrhea, and vomiting. Laboratory findings show an increased free T4 level and/or a decreased TSH level. Initial therapy is aimed at reducing the peripheral effects of thyroid hormone with β-blockers such as propranolol 80 to 120 mg po every 4 to 6 hours. The next objective in thyroid storm is to inhibit hormonal synthesis of thyroid by administering 653 propylthiouracil (PTU) 400 to 600 mg initially, then 400 to 600 mg po every 8 hours. Last, iodide is given to inhibit the release of stored thyroid hormone. Iodide is typically given as sodium iodide 250 mg IV every 6 hours or as potassium iodide (SSKI), 5 gtt po every 8 hours. Always administer PTU 1 hour before the iodide to prevent the oxidation of iodide and its incorporation in the synthesis of additional thyroid hormone

. A 51-year-old man presents to the ED with complains of colicky right upper quadrant and epigastric pain radiating to his back about 3 hours after eating. Which of the following would be the diagnostic study of choice? (A) helical CT with rectal contrast (B) right upper quadrant ultrasound (C) chest CT with IV contrast (D) acute abdominal plain radiographs (E) cardiac stress test with echocardiogram

(B) Ultrasound has been established as the most appropriate initial diagnostic imaging study in the ED for suspected biliary tract disease. Imaging studies for evaluation of biliary tract disease have parameters of the following specificity and sensitivity (cholethiasis): abdominal CT has a sensitivity of 91% and specificity of 97%; sensitivity and specificity ultrasound have been established at 91% and 97%, respectively; plain films are noted to have a sensitivity of 64% and specificity of 68%.

Which of the following is the most common etiology of upper gastrointestinal (GI) bleeding? (A) diverticulosis (B) erosive gastritis (C) Mallory-Weiss syndrome (D) peptic ulcer disease (E) inflammatory bowel disease

(D) The most common etiology of upper GI bleeding (60%) is peptic ulcer disease. This includes gastric, duodenal, and stomal ulcers. Diverticular bleeding usually results from erosion into a penetrating artery of the diverticulum. The GI bleeding associated with diverticular bleeding is usually painless and profuse. Recurrent episodes of retching can cause longitudinal tears in the cardioesophageal portion of the stomach. GI bleeding associated with this tear is known as Mallory-Weiss syndrome.

A 39-year-old woman presents to the ED with agitation, tremors, visual hallucinations, fever, and tachycardia. The eye examination reveals nystagmus and a sixth cranial nerve palsy. Which of the following conditions best describes this clinical scenario? 635 (A) Korsakoff psychosis (B) Wernicke encephalopathy (C) acute dystonia (D) acute cocaine toxicity (E) trigeminal neuralgia

(B) Wernicke encephalopathy is a potentially fatal neurologic disorder found in alcoholics with poor nutritional status that is caused by chronic vitamin B6 deficiency. Alcoholism interferes with gastrointestinal absorption of vitamin B6 and impairs conversion of vitamin B6 to its active metabolite. In many patients, concomitant liver disease impairs storage of vitamin B6. The administration of glucose to an alcoholic patient with an inadequate supply of thiamine may precipitate this disorder. *Clinical features include the triad of abnormal mental status, ophthalmoplegia, and gait ataxia.* Patients are often disoriented, forgetful, and unable to recognize familiar objects. With prompt therapy, the ophthalmoplegia usually resolves within hours and the coma resolves in hours to days, but the memory deficit may never resolve. Thiamine 100 mg administered intravenously is the treatment of choice. Thiamine 100 mg intravenous administration is continued daily until the patient has achieved proper oral nutritional status. *It is essential that thiamine be given prior to the administration of glucose*

Which of the following patient population is at the greatest risk for an epidural 625 hematoma (EDH) following head trauma? (A) infants (B) young adults (C) young children (D) elderly patients

(B) Young adults are the population at the greatest risk for epidural hematomas (EDHs) following head trauma. Direct force on the skull's temporal and parietal bones fracture can lead to lacerations of the middle meningeal artery or the dural sinus. These specific vessel lacerations account for 80% of the incidence of EDH. Subsequently, arterial hemorrhage occurs, leading to blood clotting in the space between the skull's inner table and the dura. This specific type of head injury rarely occurs in the elderly population because of the anatomical close attachment of the dura to periosteum of the inner skull's table. The dura is also closely adhered in the pediatric skull with infrequent occurrences of EDH seen in children younger than 2 years

A 58-year-old man presents to the ED with palpitations and chest pain. The ECG reveals a narrow complex tachycardia at 180 beats per minute (bpm). Which of the following medications is the most appropriate therapeutic agent for this scenario? (A) amiodarone 615 (B) lidocaine (C) adenosine (D) alprazolam (E) bretylium

(C) Adenosine 6 mg, rapid IV push over 1 to 3 seconds, is the first-line drug for paroxysmal supraventricular tachycardia. If conversion to NSR is unsuccessful after 1 to 2 minutes, an additional dose of adenosine 12 mg, rapid IV over 1 to 3 seconds, may be given. The first-line therapy for VT is lidocaine. If the patient is free of carotid bruits, vagal maneuvers such as carotid sinus massage may be tried. In patients with ischemic heart disease, ice water immersion could be detrimental and should be avoided. (Cummins; AHA, 1-32-1-35)

An 89-year-old female patient from a nursing home presents to the ED with abdominal pain and distention. The abdominal radiograph demonstrates multiple air-fluid levels and dilated large bowel loops consistent with a large bowel obstruction (LBO). What is the most likely cause of the obstruction? 618 (A) diverticulitis (B) abdominal wall hernias (C) carcinoma (D) sigmoid volvulus (E) adhesions

(C) Carcinoma of the colon is the most common cause of LBOs in adults. Diverticulitis can also cause LBOs, and patients often give a history of intermittent left lower quadrant pain. Sigmoid volvulus is a less common cause of LBO. It is seen most often in the elderly with poor bowel habits and chronic constipation.

Of the following ECG findings, which is most likely associated with hypokalemia? (A) shortened QT interval (B) ST-segment elevation (C) flattened T waves (D) prolonged PR interval (E) tall-peaked T waves

(C) Hypokalemia is associated with serum potassium levels of less than 3.5 mEq/L. The ECG findings of hypokalemia include the following: flattened T waves, U waves, and ST-segment depressions. Hyperkalemia relates to serum potassium levels of more than 5.5 mEq/L. Serum potassium levels in the 6.5 to 7.5 mEq/L typically result in tall peaked T waves and short QT interval. Prolonged PR interval is seen with 7.5 to 8.0 mEq/L serum potassium level. QRS widening, flattening of the P wave, is associated with serum potassium levels of 10 to 12 mEq/L—prolonged QT interval is also associated with hypocalcemia (Londner et al., 2004, pp. 172-174)

A 16-year-old man presents with fever, sore throat, malaise and fatigue, and cervical adenopathy for 5 days. His primary care physician (PCP) had placed him on an antimicrobial medication 2 days ago. The adolescent now presents to the ED with a diffuse widespread maculopapular rash and petechiae on the soft palate. Which of the following medications was likely prescribed by the PCP and caused the reaction? (A) acyclovir (B) azithromycin (C) amoxicillin (D) rifampin (E) tetracycline

(C) In acute mononucleosis infection, there is a 5% incidence of an associated generalized erythematous maculopapular rash. In addition, there may be petechiae on the soft palate. When patients are treated with ampicillin or an ampicillin-based antibiotic, the incidence of the rash reaches almost 100%. As the underlying etiology of mononucleosis is the Epstein-Barr virus, the treatment should be mainly supportive.

In addition to dental referral, which of the following represents the most appropriate standard therapy for a routine periodontal abscess in the ED setting? (A) intravenous penicillin and topical analgesics (B) intravenous penicillin and incision and drainage (I/D) (C) oral penicillin and oral analgesics (D) oral penicillin and saline rinses (E) oral clindamycin and topical analgesics

(C) In the emergency medicine setting, treatment of small dental abscess or periapical abscess with oral antibiotics is warranted. The most appropriate antimicrobial agents include Penicillin VK 500 mg PO QID, clindamycin 300 mg PO QID, or erythromycin 500 mg QID. Small periodontal abscess may respond to antibiotic therapy as described earlier along with the application of warm saline rinses. Larger abscesses warrant incision and drainage. It is crucial to provide sufficient analgesic therapy for dental abscesses. Analgesic therapy may include NSAIDs and/or short courses of opioid medications. Definitive therapy for dental abscesses is provided by a dentist.

Which of the following is the most important treatment option in a patient with moderate acute mountain sickness (AMS)? (A) oxygen therapy (B) dexamethasone (C) hyperbaric therapy (D) acetazolamide (E) immediate descent

(E) The three principles of treatment regarding acute mountain sickness (AMS) are (1) to stop the ascent, (2) to descend to lower altitude, and (3) to treat immediately in the presence of change in normal mental status, ataxia, or pulmonary edema. Emergent treatments include oxygen, acetazolamide, nifedipine, dexamethasone, hyperbaric therapy, and continuous positive airway pressure.

A 41-year-old man injures his finger while playing basketball. He is unable to extend the distal interphalangeal (DIP) joint. The radiograph shows an avulsion fracture to the proximal dorsal region of the distal phalanx. What is the diagnosis? (A) Bennett fracture (B) Rolando fracture (C) mallet finger fracture (D) swan neck deformity

(C) Mallet finger is a disruption of the distal tendon, resulting in a flexion deformity at the Distal interphalangeal joint (DIP). It is the most common 661 zone I injury. Bennett fracture is a combination of a dislocated carpometacarpal joint and the thumb's metacarpophalangeal joint (MCP) that is fractured intra-articularly. Rolando fracture is defined as a comminuted fracture involving the base of the thumb's MCP. A Boutonniere deformity involves deformity of the index finger. The swan neck deformity does not represent an acute finding but rather is associated with an untreated mallet finger

Which of the following is considered the most specific myocardial injury enzyme marker? (A) CK-MB (B) myoglobin (C) troponin I (D) troponin T (E) lactate dehydrogenase (LDH)

(C) Myocardial necrosis can be identified with a great degree of specificity by troponin I, particularly in settings of cocaine use, recent surgery, and chronic renal failure. Troponin T and serum myoglobin have a very high sensitivity, but the specificity is poor with regard to myocardial injury. (Green and Hill, 2004, pp. 336-338)

A 30-year-old male patient presents to the ED with an acute change in mental status. The examination reveals a patient who is sleepy but arousable to loud verbal stimuli. His airway is intact and the vital signs are stable. Investigative studies indicate an alcohol level of 150 mg/dL, an anion gap of 30 meq/L, a metabolic acidosis, an osmolar gap of 20, and calcium oxalate crystalluria. What is the most likely diagnosis? (A) methanol poisoning (B) ethanol poisoning (C) ethylene glycol poisoning (D) isopropanol poisoning (E) buspirone poisoning

(C) Patients with ethylene glycol ingestion usually present with an *acute change in mental status, high anion gap metabolic acidosis, osmolar gap, and calcium oxalate crystals in the urine.* Ethylene glycol is commercially available as preservatives, glycerine substitutes, and antifreeze. Ethylene glycol may be ingested in suicide attempts, accidentally by children, and by alcoholics as an alcohol substitute. The toxic metabolites formed by ethylene glycol metabolism are primarily formaldehyde, formic acid, and oxalic acid. When noteworthy acidosis is present, ethanol is most likely not the underlying source of intoxication. Isopropyl alcohol ingestion usually has abnormal anion gap. Methanol is noted to have a delay in presentation of toxic-related symptoms

A 46-year-old patient was diagnosed 8 days ago with a DVT of his right leg. He now presents with a white leg with absent dorsalis pedis and posterior tibial pulses. Which of the following is the most likely diagnosis? (A) phlegmasia alba dolens (B) phlebitis areta (C) phlegmasia cerulea dolens (D) phlebitis fulminans

(C) Phlegmasia alba dolens, "milk leg," is an uncommon presentation of DVT in which there is massive iliofemoral thrombosis. The leg is usually white or pale secondary to associated arterial spasm. When the dorsalis pedis and posterior pulses are diminished or absent, a false diagnosis of arterial occlusion may be made. A patient with phlegmasia cerulea dolens presents with an extensively swollen, cyanotic leg from venous engorgement due to massive iliofemoral thrombosis. This high-grade obstruction can compromise perfusion to the foot from high compartment pressures and lead to venous gangrene.

A patient presents to the ED with frequent, mucoid, watery stools, nausea, and lower abdominal pain. The patient has been on a cephalosporin-type antibiotic for about 3 months. The most likely diagnosis would be (A) diverticulitis (B) anal fissures (C) pseudomembranous enterocolitis (D) anorectal tumor (E) ulcerative colitis (UC)

(C) Pseudomembranous enterocolitis is an inflammatory bowel disorder caused by Clostridium difficile. The disorder is associated with antibiotic use and is marked by membranelike plaques of exudates that overlie and replace necrotic intestinal mucosa. The use of broad-spectrum antibiotics, notably clindamycin, cephalosporins, and ampicillin/amoxicillin, is a common cause of C difficile colonization. The treatments of choice include supportive measures and antibiotics such as metronidazole and vancomycin. Inflammation of the diverticulum, or diverticulitis, a common disorder of industrialized nations most commonly presents as pain and may be associated with changes in bowel habits. Crohn disease usually presents with abdominal pain, decrease in appetite, fever, and diarrhea; the ileum is the most commonly affected region of the bowel. Ulcerative colitis (UC) is a progressive chronic inflammatory bowel condition that usually affects the colon. UC has a variable presentation and is frequently associated with rectal bleeding. With the exception of pseudomembranous enterocolitis, none of the preceding diseases are precipitated by the use of antimicrobial agents

A 76-year-old woman (60 kg) with organic brain syndrome presents to the ED with a serum sodium level of 180 mg/dL. What is the approximate calculation of the water deficit in this hypernatremic patient? (A) 4 L (B) 6 L (C) 8 L (D) 11 L (E) 14 L

(C) The definition of hypernatremia is a serum sodium level more than 145 mEq/L. Hypernatremia is classified as isovolemic (diabetes insipidus, skin loss through hyperthermia, and iatrogenic), hypervolemic (administration of hypernatremic solutions, mineralocorticoid excess as in Conn or Cushing syndrome, and salt ingestion), and hypovolemic (renal losses through diuretics or glycosuria, GI, respiratory, or skin losses, and adrenal deficiencies). Water deficit in hypernatremic patients is calculated by the following formula: In Women: Water deficit (L) = [0.5 (body weight, kg)] × (measured serum sodium/normal serum sodium) − 1

The most immediate management priority in a patient with septic shock is: (A) empiric antimicrobial therapy (B) inotropic support (C) oxygenation and ventilation (D) fluid therapy (E) acid-base status

(C) The first priority in the management of septic shock is assessment of the airway, oxygenation, and ventilation. Oxygen should be administered at 100% via mask or endotracheal tube. Fluid resuscitation is the second priority in the patient with septic shock. Tissue and organ perfusion can be assessed by parameters such as the patient's mental status, blood pressure, respiratory rate, pulse rate, skin color and temperature, central venous pressure, and urine output more than 30 mL/h (1 mL/kg/h in pediatric patients). Other important areas of assessment and management include acid-base status and antimicrobial therapy. (Jui, 2004, pp. 236-241)

Which of the following extrapulmonary findings is most likely to be associated with M pneumoniae? (A) bullous myringitis (B) conjunctivitis (C) lymphangitis (D) otitis externa (E) guttate psoriasis eruption

. (A) Mycoplasma pneumonia is most prevalent in older children, young adults, and the elderly. Bullous myringitis, rash (erythema multiforme), neurologic symptoms, arthritis, and arthralgia are common extrapulmonary symptoms found in patients with Mycoplasma pneumonia

A 16-year-old long-distance runner suffered an external rotation injury to the ankle. Which of the following ligaments is most likely injured? (A) anterior talofibular (B) posterior talofibular (C) deltoid (D) calcaneofibular (E) tibiofibular

(C) The medial ankle support comprises the deltoid ligaments, which include tibionavicular, anterior tibiotalar, tibiocalcaneal, and posterior tibiotalar parts. The most common mechanism of injury is an external rotational force. The lateral portion of the ankle is supported by the anterior talofibular, anterior inferior tibiofibular, interosseous, posterior tibiofibular, and the calcaneofibular ligaments. Approximately two-thirds of all ankle injuries are isolated anterior talofibular ligament injuries. About 20% involve both anterior talofibular and calcaneofibular ligament injuries. Less than 5% of the ankle injuries are isolated deltoid ligament sprains. (Ho and Abu-Laban, 2006, pp. 808-820)

A 72-year-old woman is brought to the ED after being found on her kitchen floor comatose with a BP of 280/150 mm Hg and pinpoint reactive pupils. What is the most likely diagnosis? (A) thalamic hemorrhage (B) cerebellar hemorrhage (C) pontine hemorrhage (D) subarachnoid hemorrhage (E) intracerebral left occipital hemorrhage

(C) The typical presentation of pontine hemorrhage is coma, pinpoint reactive pupils, impaired lateral ocular motility, and quadriplegia with decerebrate posturing. Thalamic hemorrhage characteristically leads to impaired consciousness, contralateral motor and sensory loss, and gaze preference to the side of the hemorrhage. Cerebellar hemorrhages typically present with impaired gait, vertigo, limb ataxia, impaired consciousness, and cranial nerve palsies. Subarachnoid hemorrhage presents as a severe acute onset of a headache with associated neurological deficits of varying degrees depending on the extent and location of the hemorrhage. Lobar hemorrhages may present with symptoms of headache, nausea, vomiting, change in mental status, visual disturbance, seizures, and coma, depending on the extent and location of the bleed. (Zivin, 2007, pp. 2769-2775)

A 29-year-old logger was struck in the back with a load of logs. The evaluation reveals absence of patellar reflexes. This finding is consistent with an injury at which of the following dermatome levels? (A) C7 (B) L1 (C) L4 (D) S1 (E) S4

(D) An L4 injury may present as weakness or paralysis to the quadriceps and thigh adductor muscles, sensory loss to the medial leg, and loss of the patellar reflexes. An injury to C7 may manifest as decreased sensation in the middle finger and the loss of the triceps reflexes and thumb extension. An injury to L1 would involve loss of the cremasteric reflex, decreased sensation in groin area, and loss of hip flexion. An injury to S1 would more likely be indicated by loss of sensation of the lateral dorsal and plantar aspect of the foot, loss of Achilles reflex, and loss plantar flexion. S4 deficits may present with a loss of the anal reflex (wink), loss of perineal area sensation, and loss of voluntary control of the pelvic floor.

A 42-year-old man presents to the ED with a right-sided facial injury after an assault with a wooden club. The patient complains of diplopia and pain to the right side of the face. The examination reveals enophthalmos, impaired ocular motility, and infraorbital hypoesthesias. What is the most likely diagnosis? (A) maxilla fracture involving the superior orbital ridge (B) orbital blowout fracture with herniation of contents into the frontal sinus (C) maxillary blowout fracture with herniation into the soft palate (D) orbital blowout fracture with herniation of contents into the maxillary sinus (E) orbital blowout fracture without herniation

(D) Direct and compressive forces to the eye may cause a blowout fracture to the orbital floor with herniation of the contents into the maxillary sinus. Blowout fractures may produce enophthalmos, diplopia, impaired ocular motility, and infraorbital hypoesthesias. Many orbital floor fractures resolve spontaneously and require only close follow-up with consultants. A decision to operate may be delayed 10 to 14 days, depending on persistent diplopia or enophthalmos.

Which of the following drugs represents the most appropriate antidotal agent for benzodiazepine overdose? (A) naloxone (B) activated charcoal (C) ketamine (D) flumazenil (E) flutamide

(D) Flumazenil competitively blocks the effects of benzodiazepines on GABAnergic pathway-mediated inhibitors in the central nervous system (CNS). Naloxone HCl (Narcan) is a narcotic antagonist. Ketamine is a rapidacting general anesthetic. Flutamide is a nonsteroidal, antiandrogenic agent used for prostate carcinoma. (Bosse, 2004, pp. 1055-1056)

A 6-year-old boy presents to the ED with abdominal pain, blood in the stools, and arthritis. The examination reveals multiple dark erythematous lesions on his legs and buttocks. These findings are characteristic of which disease? (A) Kawasaki disease (B) meningococcemia (C) erythema nodosum (D) Henoch-Schonlein purpura (E) pneumococcal meningitis

(D) Henoch-Schonlein purpura is characterized by the triad of abdominal pain, arthritis, and nonthrombocytopenic purpura. This condition is typically immunologically mediated vasculitis by a stimulus that often cannot be identified. However, at least half of the affected patients had an upper respiratory tract infection. Occasionally, a drug or bacterial agent can be identified as the source. Patients characteristically develop a slightly raised symmetrical petechial rash, which is most prominent on the lower extremities. In addition, patients develop colicky abdominal pain, bloody diarrhea, intussusception, and migratory large joint arthritis. Renal manifestations may present in the form of hematuria, proteinuria, and nephrosis. Therapy is directed at identifying and treating the underlying problem. Kawasaki disease is an inflammatory vasculitis characterized by fever for 5 days and four out of five of the following criteria: conjunctival injection (bilateral), strawberry tongue and mouth fissures, desquamation and swelling of the fingers and toes, erythematous rash starting on the palms and soles, and enlarged lymph nodes. Erythema nodosum is an inflammatory disease of the skin and sub-cutaneous tissue characterized by tender red nodules. Etiologies include bacterial infections (deep fungal infections, e.g., histoplasmosis, coccidioidomycosis), 659 viral upper respiratory tract infection drugs, and idiopathy. The rash is described as multiple raised, warm, tender nodules with bluish discoloration that most commonly involve the pretibial region; however, the forearms or thighs may be involved as well. Pneumococcal meningitis does not typically present with a petechial rash or purpura. Meningococcemia is caused by encapsulated gram-negative diplococci that present with 1 to 2 mm petechiae to full-blown ecchymoses

Air contrast enema is diagnostic in approximately what percentages of intussusception cases of less than 24 hours of duration? (A) 1% (B) 10% (C) 30% (D) 70% (E) 95%

(D) In evaluating a pediatric patient for suspected intussusception, air contrast enema has proven to be diagnostic and therapeutic in approximately 60% to 80% of cases. Contrast enema utilizing air is preferred to barium because of greater management of colonic pressures when performing the reduction when compared with the barium technique. An additional benefit in the case of bowel perforation is that there is no risk of spillage of barium contrast into the peritoneum. The patient should be stable and well resuscitated before undergoing the contrast enema procedure.

Which of the following is the most appropriate indication for IV thrombolytic therapy in a patient with acute coronary syndrome? (A) 0.5 mm ST-segment elevation in the inferior wall leads resolving with two sublingual nitroglycerins (B) posterior wall myocardial infarction duration of 12 to 14 hours (C) right bundle branch block (D) 2 mm of ST-segment elevation in the anterior/lateral wall (E) 3 mm of ST-segment depression

(D) Indications for IV thrombolytic therapy include acute myocardial infarction less than 6 to 12 hours old, new or presumed new left bundle branch block, and an ECG that has at least 1 mm of ST-segment elevation in two or more contiguous leads. Additional indications include chest pain and ST elevation unresolved with nitroglycerin. ST-segment depression is not an indication for thrombolytic therapy. Known absolute and relative contraindications to use of thrombolytics must always be taken in consideration. (Hollander and Dierks, 2004, pp. 352-353; Humphries, 2008, pp. 561-562)

A 47-year-old man presents to the ED comatose after ingesting an unknown liquid substance. Investigative studies include the following: pH, 7.45; Na, 140; Cl, 110; HCO3, 19; glucose, 180; BUN, 30; Cr, 1.5; ETOH, 0.0; high serum ketones; and a measured osmolality of 380. These findings are most consistent with which of the following toxin ingestions? (A) methanol (B) ethylene glycol (C) diabetic ketoacidosis (D) isopropanol (E) alcoholic ketoacidosis

(D) Isopropyl alcohol (isopropanol), commonly referred to as rubbing alcohol, is a solvent and disinfectant used in many household items such as hair and skin products, antifreeze, and window cleaning solutions. The toxic dose is 1 mL/kg of a 70% solution. Isopropanol is metabolized to acetone. Mild acidosis may occur because of the formation of acetate and formate. This toxicity is associated with high serum and urine ketone levels. However, a distinguishing factor is that there is no increase in the osmolal gap or anion gap acidosis. Methanol, also referred to as wood alcohol, is commonly used in products such as solvents, antifreeze, windshield washer fluid, and varnishes. The lethal ingested dose is approximately 15 to 30 mL in adults. Methanol is oxidized in the liver to formaldehyde and formate, subsequently a severe lactic acidosis develops. These metabolites concentrate in the vitreous humor and optic nerve, causing ocular toxicity and blindness. This toxicity is associated with an osmolal gap and an anion gap acidosis but no ketosis. Ethylene glycol is frequently implicated in overdoses of antifreeze. Clinical findings include increase in serum potassium level and the presence of a wide anion gap metabolic acidosis. Diabetic Ketoacidosis (DKA) is a disorder found in 664 insulin-dependent diabetes patients that is characterized by hyperglycemia, ketonemia, and acidosis. Serum glucose levels are typically higher than 300 mg/dL. Metabolic acidosis is demonstrated by a serum bicarbonate concentration of less than 15 mEq/L and a pH of less than 7.2. Ketonemia results from β-hydroxybutyrate and acetoacetate. This toxicity results in an anion gap acidosis, a ketotic state, but no osmolal gap. Alcoholic ketoacidosis is typically seen in alcoholic patients who are forced to stop drinking shortly after a drinking binge. β-Hydroxybutyric acid is the predominant ketone formed in alcoholic ketoacidosis. A metabolic acidosis may occur from vomiting, dehydration, and respiratory alkalosis. Therefore, this toxicity is characterized by an anion gap acidosis and a high ketone level but an osmolal gap is not found

A 25-year-old patient presented to the ED with a 3 cm linear left palmar hand laceration from a broken glass bottle container. Which of the following would be the most appropriate initial management of the hand injury? (A) Allow the wound to close by secondary intention after irrigation. (B) Explore the wound and place the patient on prophylactic antibiotics. (C) Obtain a hand radiograph, explore wound, and suture the wound if no foreign body is observed. (D) Obtain a hand radiograph and consult a hand surgeon. (E) Suture the wound and place the patient on prophylactic antibiotics.

(D) Organic material is generally not visible on plain radiographs. Glass and metal that are a reasonable size of at least 2 mm should be visible on plain radiographs. After imaging and wound exploration, if no foreign body is located, it is highly unlikely to be present in the wound. If the wound was at high risk of infection, a delayed wound closure would be indicated. (Hollander and Singer, 2004, p. 287-288)

Which of the following clinical findings differentiates periorbital from orbital cellulitis? (A) erythema (B) fever (C) lid edema (D) worsening pain with eye movements (E) development of a rash on the face

(D) Periorbital cellulits is characterized by warmth, redness, swelling, and tenderness over the affected eye, along with conjunctival injection, eyelid swelling, chemosis, and fever. Orbital cellulitis includes all the symptoms of periorbital (preseptal) cellulitis with the addition of ocular pain and limitation of eye movement. Other physical examination findings may include lid edema, proptosis, marked tenderness to the globe, decreased visual acuity, and pupillary paralysis.

In community-acquired pneumonia, which of the following bacterial organisms is most likely to be implicated? (A) Klebsiella pneumoniae (B) Mycoplasma pneumoniae (C) Neisseria meningitidis (D) Streptococcus pneumoniae (E) Pseudomonas aeruginosa

(D) Pneumococcus is the most commonly implicated bacteria in community- acquired pneumonia, except for Mycoplasma pneumonia, which is classified as an atypical pneumonia. The other organisms are not frequently classified as community acquired

A 3-month-old infant presents with low-grade fevers, rhinorrhea, cough, wheezing, mild retractions, and no difficulty feeding for 3 days. The oxygen saturation was greater than 93%. The medical history is noted for 35 weeks' gestation at birth. Immunizations are up to date. Otherwise, the infant has been healthy. The infant was diagnosed with respiratory syncytial virus (RSV). Which of the following clinical rationales would most likely warrant hospital admission? (A) evaluation for additional coexisting respiratory infections (B) intravenous antimicrobial therapy (C) intravenous steroid therapy (D) monitoring for episodes of apnea or respiratory failure (E) parent education

(D) RSV pulmonary infections in young infants can be accompanied by apneic episodes as well as chlamydia and pertussis infections. Mucous plugging results from necrosis of the respiratory epithelium and destruction of ciliated epithelial cells. This and submucosal edema lead to peripheral airway narrowing and variable obstruction mechanism inducing RSV-related apnea in young infants is not completely understood but may be related to hypoxemia and upper airway obstruction. Infants at the highest risk are those younger than 668 6 weeks and those who have a history of prematurity, apnea of prematurity, and low O2 saturation on admission. It is difficult to predict apneic events. Steroids are not recommended as studies have failed to prove benefit unless underlying asthma; in addition, they can be administered in the oral formulation. Hospital admission is recommended for children with clinically defined hypoxia and RSV diagnosis; the frequently chosen parameter is pulse oximetry of less than 90% to 93%. Ribavirin (antiviral) is clinically indicated for RSV infections in high-risk patients and comes in oral and aerosol formulations. Educating the parents on the clinical course of RSV infection and signs and symptoms or respiratory distress is critical in disposition of pediatric patients. Children with mild symptoms who are tolerating fluids can be released in the care of capable caregivers with good follow-up. Additional home care options can include home health nursing visits and discharging the patient with nebulizer machines if medically necessary

Which of the following signs and symptoms is most likely to be indicative of acute myocardial ischemia or infarction? (A) dyspnea (B) left arm numbness and tingling (C) referred back pain (D) retrosternal chest discomfort (E) sternal chest pain

(D) Retrosternal chest discomfort that is best described as pressure or squeezing as opposed to a pain is most likely to indicate myocardial ischemia or infarction.

A tall, thin, 26-year-old woman presents to the ED with an acute onset of right- 620 sided pleuritic chest pain and dyspnea. What is the most likely diagnosis? (A) right tension pneumothorax (B) left pneumothorax (C) right pleural effusion (D) right spontaneous pneumothorax (E) left atelectasis

(D) Spontaneous pneumothorax most commonly affects tall, thin men, between the ages of 20 and 40 years, who are heavy cigarette smokers. The pain is usually pleuritic and localizes to the affected side. Most patients have decreased breath sounds on the affected side, but few have a significant tachypnea or tachycardia

What is the mechanism of injury for an anterior shoulder dislocation? (A) shoulder internal rotation and adduction (B) shoulder internal rotation with abduction (C) shoulder external rotation with adduction (D) shoulder external rotation with abduction (E) fall on an outstretched hand

(D) The most common mechanism of injury for an anterior shoulder dislocation is abduction, extension, and external rotation. The lateral edge of the acromion process is prominent and the arm is held in slight abduction and external rotation by the opposite extremity. Anterior shoulder dislocations account for 95% to 97% of all glenohumeral dislocations. Falls onto an outstretched hand as mechanism of injury is more common in older patient population.

A 19-year-old woman presents to the ED with a 4-day history of a warm, swollen, erythematous hand following a cat bite. Which of the following is the most likely causative pathogen in this scenario? (A) Staphylococcus intermedius (B) Haemophilus aphrophilus (C) Eikenella corrodens (D) Pasteurella multocida (E) S aureus

(D) The most common pathogen found in infected cat bite wounds is P multocida. Pathogens like S intermedius, E corrodens, and S aureus are commonly seen in infected dog bite wounds. Infection with Capnocytophaga canimorsus infection is a rare occurrence secondary to dog bites that can lead to a widespread virulent bacterial infection. H aphrophilus infection is more common after tongue piercings. (Schwab and Powers, 2004, p. 327)

A 68-year-old woman presents to the ED with an exacerbation of chronic low- back pain. Which of the following indicates the patient has developed cauda equina syndrome? (A) lower leg weakness, paresthesias to both legs, and incontinence (B) loss of deep tendon reflexes bilaterally and urinary retention (C) bilateral leg weakness, loss of peripheral pulses, and incontinence (D) bilateral leg pain, saddle anesthesia, urinary incontinence, and fecal incontinence (E) anesthesia to entire leg, bilateral leg weakness, and loss of deep tendon reflexes

(D) The most severe neurological dysfunction, as a result of inadequate or delayed treatment of disk herniation, is cauda equina syndrome. The most common presenting symptoms are saddle anesthesia, bilateral leg pain, urinary incontinence or retention, and fecal incontinence or retention. Most cases of cauda equina syndrome and cord compression develop over a matter of hours. If the symptoms are delayed, these patients are at high risk for chronic neurological deficits.

A 28-year-old man presents to the ED with a back injury following an all- terrain vehicle (ATV) crash. The examination reveals a sensory deficit at the nipples. The spinal cord injury level is most likely at which of the following levels? (A) T1 (B) T2 (C) T3 (D) T4 (E) T5

(D) The thoracic T4 dermatome runs across the nipple line. T2 dermatome involves the upper medial bicep region. T3 runs just above the nipple line, and T5 runs just below the nipple line

A 20-year-old man presents to the ED following a lethal overdose of acetaminophen. What is the antidote for acetaminophen toxicity? (A) flumazenil (B) narcan (C) vitamin K (D) N-acetylcysteine (NAC) (E) ethanol

(D) Treatment priorities of acetaminophen toxicity consist of supportive care, gastrointestinal decontamination, and the use of the antidote N-acetylcysteine (NAC). No additional therapies are recognized for intervention in acetaminophen overdoses. If given early (less than 8 hours after ingestion), NAC can prevent toxicity by inhibiting the binding of the toxic metabolite N- acetyl-p-benzoquinoneimine to hepatic proteins. In acetaminophen toxicity, more than 24 hours after ingestion, NAC diminishes hepatic necrosis by nonspecific mechanisms. The standard 72-hour oral NAC regimen used in the United States is a loading dose of 140 mg/kg followed by maintenance doses of 70 mg/kg every 4 hours for 17 doses.

Which of the following is the most compelling indication for thrombolytic therapy in a patient with acute pulmonary embolism? 617 (A) severe dyspnea (B) poor tissue perfusion (C) right ventricle dysfunction (D) circulatory collapse and refractory hypoxemia (E) severe right heart pressures

(D) Use of thrombolytic agents in the clinical setting of pulmonary embolism is steep in controversy. In evaluation of the benefits to risk ratio, the situation of a massive pulmonary embolism associated with sustained severe hypotension characterized as a systolic pressure of less than 80 mm Hg, refractory hypoxemia, and circulatory collapse should receive a thrombolytic agent such as alteplase. Thrombolytics have proven to be effective in normalizing pulmonary artery pressures, improving right ventricular dysfunction, stabilizing hemodynamics, and correcting hypoxia. The use of fibrinolytic agents such as urokinase and streptokinase and alteplase have been proven to be effective and are approved by the U.S. Food and Drug Administration

Using a scoring system such as the Pneumonia (PORT) Severity Index to determine the risk of mortality, which of the following patients could be appropriately managed with outpatient treatment for community-acquired pneumonia? (A) 49-year-old male nursing home resident with congestive heart failure, a serum blood urea nitrogen (BUN) level of 35 mg/dL and bilateral pleural effusions (B) 51-year-old woman with breast cancer, a temperature of 41°C, and blood glucose level of 260 mg/dL (C) 75-year-old woman with liver disease and altered mental status (D) 86-year-old man with a respiratory rate of 32/min (E) 86-year-old woman with a respiratory rate of 28/min and a history or chronic renal insufficiency

(D) Using the Pneumonia PORT, severity criteria points are assigned by gender, age, nursing home resident, underlying disease, and alterations of hemodynamic and abnormal diagnostic findings. Scores greater than 91 tend to have higher risk of mortality with recommendations to hospitalize

A 64-year-old woman presents to the ED with complaints of left-sided headache, low-grade fever, malaise, pain with chewing, and decreased vision in her left eye. Which of the following would be the most appropriate measure in managing this patient? (A) aspirin therapy (B) lumbar puncture (C) muscle relaxants (D) intravenous steroids (E) oxygen (O2) therapy

(D) temporal arteritis or giant cell arteritis. The most appropriate course of action is to treat with intravenous steroids and obtain ophthalmology consultation when patient has visual loss.

A patient presents to the ED after suffering a significant pelvic injury following a fall. The patient has a grade III pelvic fracture and blood at the tip of the urethral meatus. How should one proceed in evaluating urethra and/or bladder injuries? (A) Gently pass a 14- or 16-Fr Foley catheter. (B) Gently pass a 14- or 16-Fr Coudé catheter. (C) Notify the urologist for immediate cystoscopy. (D) Gently pass a 10- or 12-Fr (pediatric) Foley catheter. (E) Perform a retrograde urethrogram.

(E) A retrograde urethrogram should be performed prior to invasive interventions such as urethral catheterization if there is any possibility of urethral disruption. Signs of urethral disruption include a high riding prostate and the presence of blood at the tip of the meatus. Sixty milliliters (or 0.6 mL/kg) of full-strength or half-strength iothalmate meglumine (Conray II) is injected over 30 to 60 seconds. A radiograph is taken during the last 10 mL of contrast material. Retrograde flow through the urethra and into the bladder without extravasation ensures continuity of the urethra and absence of urethral injury.

A patient is being evaluated for cough, fever, night sweats, and sputum production. On the basis of the history, the patient is suspected to have active tuberculosis. Which of the following is most significant in the ED setting for management of a patient with suspected active TB infection? (A) hospital admission for multidrug resistant tuberculosis (MDRTB) (B) respiratory isolation in negative pressure room (C) health care providers should wear N-95 particulate respirators or mask (D) reporting to local public health department (E) all of the above

(E) Active multidrug resistant TB (MDRTB) warrants hospital admission. Limiting exposure to these patients, who are likely infectious, is best accomplished by early identification and placement in a negative airflow room (respiratory isolation). Staff working in the ED should be accustomed to using respiratory protective equipment, specifically protective masks, with the more advanced staff using the N-95 particulate respirators.

A 79-year-old man presents to the ED with a heart rate of 50 bpm, second degree A-V block Mobitz at 50 bpm, BP of 90/60 mm Hg, potassium level of 5.8 mEq/L, and a digoxin level of 6.9 ng/mL. What is the treatment of choice in this scenario? (A) calcium chloride (B) potassium infusion (C) atropine (D) magnesium (E) digoxin immune Fab fragments

(E) Digoxin toxicity results in enhanced excitability and contractility in myocardial muscle and decreased conduction velocity in conduction tissue. Therapy is directed at airway management, cardiovascular resuscitation, and continuous cardiac monitoring. Ventricular ectopy is treated with lidocaine or phenytoin, and symptomatic bradycardia is treated with atropine. Antidote treatment is with digoxin-specific antibodies, which consist of Fab fragments that bind digoxin, removing it from cardiac receptors and reversing toxicity. Indications include life-threatening dysrhythmias, a serum potassium level higher than 5.0 mEq/L, a serum digoxin level higher than 10 to 15 ng/mL, and advanced age. Potassium replacement would be contraindicated in acute toxicity with elevation of the serum potassium level; calcium chloride may worsen the arrhythmia. Variable results such as bradycardia and heart block are noted with the use of atropine. Magnesium is best indicated for tachyarrhythmias associated with digoxin toxicity.

The most reliable clinical assessment tool to confirm endotracheal intubation is which of the following? (A) endotracheal tube condensation (B) symmetrical chest expansion (C) breath sounds auscultated equally over the chest (D) no breath sounds auscultated over the stomach (E) use of a carbon dioxide (CO2) detection device

(E) Direct visualization of the endotracheal tube passing through the vocal cords is the most reliable indicator for endotracheal tube placement. The next most reliable is an end CO2 device. Auscultation of the chest and epigastric areas may reveal transmitted sounds from the endotracheal tube in the stomach. Condensation in the tube can be an unreliable indicator of proper endotracheal tube placement.

. Which of the following is the most common cause of death by hemorrhage in patients with hemophilia A? (A) gastrointestinal hemorrhage (B) retroperitoneal hemorrhage (C) pulmonary hemorrhage (D) renal hemorrhage (E) intracranial hemorrhage

(E) Hemophilias A and B are X-linked recessive disorders that are deficiencies in factor VIII and factor IX, respectively. Approximately 85% of patients with hemophilia have hemophilia A. Severity of the disorder depends on the level of factor deficiency. Common clinical findings include hemarthroses, soft tissue bleeding, muscular hematomas, and intracranial bleeding. Intracranial bleeding is the leading cause of death in people with hemophilia. Gastrointestinal bleeding is rare.

Which of the following disorders may present with a patient experiencing anxiety, tremors, palpitations, fatigue, and hemiplegia? (A) hyperglycemia (B) euglycemia (C) ketoacidosis (D) hyperglycemic hyperosmolar nonketotic coma (E) hypoglycemia

(E) Hypoglycemia is defined as a plasma glucose level less than 50 mg/dL; however, the criteria for the diagnosis should include the presence of symptoms, low plasma glucose level in a symptomatic patient, and relief of 654 symptoms after ingestion of carbohydrates. Contributing factors to symptoms of hypoglycemia include the rate at which the glucose decreases patient's overall size, underlying health conditions, and previous hypoglycemic reactions. Common symptoms include anxiety, diaphoresis, tremors, tachycardia, palpitations, fatigue, syncope, headache, mental status changes, visual disturbances, and hemiplegia. Patients with ketoacidosis are noted to frequently have Kussmaul respirations, changes in BP, and increased respirations and heart rate. These patients may also have ketone or acetone breath odor. Patients with new-onset hyperglycemia may present with increased thirst, increased urination, and increased appetite. Euglycemia refers to normal blood glucose levels; subsequently, patient should be asymptomatic. Hyperglycemic hyperosmolar nonketotic coma (HHNC) relates to a disorder that affects elderly diabetic patients with findings of increased osmolarity, blood glucose level, and dehydration. These patients have a spectrum of changes in mentation from confusion to frank coma.

Which of the following conventional or plain radiographic views of the face is most beneficial for evaluating trauma of the midface? (A) panorex (B) submental-vertex (C) Towne view (D) Caldwell view (E) Waters view

(E) In evaluating facial trauma of the midface region, the Waters view has been established as the most sensitive of plain radiographs. The Waters view is useful in evaluation of the presence of orbital rim fractures and air-fluid levels in the maxillary sinuses. The most appropriate plain radiograph to evaluate the upper face is the Caldwell or posteroanterior view. To evaluate the base of the skull and zygoma, the submental view is the most appropriate plain radiograph. Although not available at all institutions, when available, the Panorex is the best imaging study for evaluation for suspected fracture of the 657 mandible fractures. To evaluate the mandible ramus and condyles, the Towne view is the best imaging view.

A 21-year-old woman presents to the ED with a several-hour history of abdominal pain that has localized to the right lower quadrant (RLQ), with associated nausea, vomiting, diarrhea, and anorexia. For suspected appendicitis, which of the following would be the diagnostic study of choice? (A) abdominal pelvic ultrasound (B) plain abdominal radiograph (C) abdominopelvic helical computed tomography (CT) unenhanced CT abdomen/pelvis (D) magnetic resonance imaging (MRI) (E) focused appendiceal helical CT with oral and colonic contrast CT abdomen/pelvis

(E) In evaluation of suspected appendicitis a focused appendiceal helical CT with oral and colonic contrast has a sensitivity and specificity of 100% and 95%, respectively. Abdominal pelvic ultrasound has a sensitivity and specificity of 84% and 96%, respectively. Plain films have a sensitivity and specificity of 48% and 58%, respectively. An unenhanced abdominopelvic helical CT has slightly decreased sensitivity of 91% and specificity of 95%. MRI yields sensitivity of 97% and 92% specificity but is not frequently utilized for such evaluation.

Which of the following is the most likely underlying source of an adult food bolus impaction? (A) angiodysplasia (B) bezoars (C) Boerhaave syndrome (D) Mallory-Weiss syndrome (E) Schatzki ring

(E) Partially chewed meat and esophageal abnormalities such as strictures, esophageal spasms, and Schatzki rings are commonly implicated in adult food bolus impactions.

Pseudomonas pneumonia is most likely associated with which of the following patient populations, signs, symptoms, and diagnostic findings? (A) immunocompromised patients such as alcoholics with an acute onset of fevers, rigors, and chest pain (B) minimal cough, mild fever, and minimal radiographic chest radiographic findings (C) prevalence in patients with chronic lung disease, insidious onset with low- grade fevers, dyspnea, and sputum production (D) prevalence in very young and old, high fever, bloody sputum, and chest pain (E) severe cyanosis, confusion, signs of systemic illness, and chest radiographic findings of bilateral lower lobe infiltrates and occasional empyema

(E) Pseudomonas pneumonia is commonly associated with hypoxia, confusion, fever, sepsis, and signs of systemic illness and lower lobe infiltrate on chest radiograph.

A 7-year-old child presents to the ED with fever, neck pain, and a "duck-like" voice. Which of the following is the most likely diagnosis? (A) peritonsillar abscess (B) streptococcus pharyngitis (C) epiglottitis (D) Ludwig angina (E) retropharyngeal abscess

(E) Retropharyngeal abscess is an infected fluid collection in the fascial plane between the posterior pharyngeal muscles and the paraspinous muscles. Primarily, retropharyngeal abscess is a pediatric problem because there are lymph nodes in the retropharyngeal space that can become suppurative. Clinical manifestations include an ill-appearing child with fever, sore throat, neck pain, and voice changes (ie, "duck-like voice"). A CT scan with IV contrast of the soft tissues of the neck and upper chest is the best diagnostic test. Peritonsillar abscess is an infected fluid collection in the pharyngeal pillar. The most common etiology is β-hemolytic streptococcus. Symptoms include fever, sore throat (unilateral), and odynophagia. In addition, the patient drools and finds it hard to handle his/her own secretions. Streptococcal pharyngitis is an infection of the pharynx and tonsils due to group A β- hemolytic streptococci. Clinical features include sudden onset of fever and sore throat with enlargement of the cervical lymph nodes. Headache, vomiting, abdominal pain, meningismus, and torticollis can occur as well. Epiglottitis is an inflammatory disorder of the supraglottic laryngeal region. Etiologies of 662 epiglottitis include bacterias, viruses, chemical damage (eg, aspiration of fuel), and mechanical damage (eg, trauma, burns). Symptoms include sore throat, fever, a muffled voice, dysphagia, and respiratory distress. Clinical features include drooling, dyspnea, tachypnea, inspiratory stridor, tripod position (ie, patient leans forward, supporting himself/herself with both hands), and toxic appearance. Ludwig angina is an abscess formation of the submaxillary, sublingual, and submental spaces accompanied by elevation of the tongue. The cause is due to an infection of the lower second and third molars usually due to β-hemolytic streptococcus, staphylococcus, and mixed anaerobic and aerobic infections. Patients commonly present with swelling beneath the chin. The tongue is displaced up and posteriorly. Trismus often makes opening the mouth for examination difficult.

A 62-year-old patient presents to the ED with generalized abdominal pain, nausea, vomiting, and abdominal distention. The radiographs demonstrate multiple loops of dilated small bowel, air-fluid levels, and a string of pearls sign. What is the most likely cause of this clinical scenario? (A) neoplasm (B) incarceration of abdominal hernias (C) gallstone ileus (D) bezoars (E) adhesions following abdominal surgery

(E) Small bowel obstruction (SBO) is most often due to adhesions following surgery. Incarcerated groin hernias are the second most common cause of SBOs. Other hernias that are responsible for SBOs are umbilical, femoral, and obturator foramen. Less common causes of SBOs are polyps, lymphoma, and adenocarcinoma. Bezoars (undigested vegetable matter) represent an intraluminal obstruction in those having undergone prior surgeries such as pyloric resection. Gallstone ileus is an unusual cause of intraluminal SBO. The most common cause of LBO is neoplasm.

A 68-year-old man with a medical history of coronary artery disease presents to the ED with a 2-day history of intermittent chest tightness. The pain was not relieved with three nitroglycerin sublingual tablets. What would be the initial assessment if the electrocardiogram (ECG) demonstrated 2 mm ST-segment elevations in leads II, III, and aVF? 612 (A) acute anterior wall myocardial infarction (B) acute lateral wall myocardial injury (C) subendocardial inferior wall myocardial infarction (D) subendocardial anterior wall myocardial ischemia (E) acute inferior wall myocardial injury

(E) The ECG criterion used to define an acute myocardial injury pattern is ST- segment elevation of 1 mm or more above the baseline. This should be measured 0.04 seconds past the J point. ST-segment depression and T wave changes are indicative of myocardial ischemia. Abnormal Q waves are consistent with myocardial infarction. Q waves that are 2 mm wide or 25% of the height of the R wave in that lead are indicative of dead heart muscle. The following are the anatomical sites of coronary ischemia related to the ECG leads: anterior septal wall is leads V1 to V4; high lateral wall is leads V5, V6, plus I, and aVL; inferior wall are leads II, III, and aVF; posterior wall leads demonstrate marked depression in leads V1 to V4 (mirror image of anterior wall); and right ventricular infarction will demonstrate ST-segment changes in lead V4R. (Cummins, 1999, pp. 9-30)

Which of the following clinical conditions would the focused assessment sonography for trauma (FAST) be most useful in confirming the suspected diagnosis? (A) suspected ectopic pregnancy (B) right upper quadrant pain (C) suspected retroperitoneal bleeding (D) right calf swelling and pain (E) blunt abdominal injury

(E) The focused abdominal sonography for trauma (FAST) has most notably been useful in evaluation of trauma patients utilizing the noninvasive diagnostic modality of ultrasonography. The FAST examination consists of four standard views. It is helpful in identifying those patients in need of emergent laparotomy as it is able to detect blood in the peritoneal cavity. Retroperitoneal bleeding may not be visualized with FAST evaluation. Ultrasonography is useful in evaluating patients in the first trimester of pregnancy; however, the FAST examination approach is not typically applied when assessing pathology of the extremity venous system and the right upper quadrant of the abdomen.

Which of the following signs and symptoms best describes the presentation of a dissecting thoracic aortic aneurysm? (A) syncope, abdominal pain, back pain, and shock (B) severe quadriplegia and coma (C) abrupt and severe pain in the chest or between the scapulae (D) lower back pain with radiation into the legs (E) headache, neck pain, and vomiting accompanied by an inability to walk or stand

. (C) Severe, abrupt chest pain that also may be located between the scapulas is common in aortic dissection. Pain in the anterior chest or back may represent involvement of the ascending or descending aorta. In the setting of symptoms of severe back or abdominal pain accompanied by shock, a dissecting abdominal aneurysm is strongly likely. Rapid onset of stroke, inability to move muscles 649 (except the lateral gaze), and quadriplegia are indicative of basilar artery occlusion. Headache, neck pain, and vomiting accompanied by the sudden onset of an inability to walk or stand are associated with cerebellar infarction.

A patient is involved in a motor vehicle accident and suffered a fractured neck. The fracture lines extend through the pedicles of C2. Which of the following describes this unstable hyperextension fracture to the cervical spine? (A) Jefferson fracture (B) extension teardrop fracture (C) clay-shoveler fracture (D) Johnson fracture (E) hangman fracture

. (E) An unstable, hyperextension fracture through the pedicles of C2 is known as a hangman fracture. Fortunately, cord damage is usually minimal because the anteroposterior diameter of the neural canal is greatest at the C2 level. Furthermore, less neurological damage occurs because bilateral pedicle fractures tend to decompress themselves, allowing more space for the spinal cord. A Jefferson fracture of C1 is produced by an axial loading injury to the cervical spine, transmitting a force through the occipital condyles to the superior articular surfaces of the lateral masses of the atlas. A clay-shoveler fracture is an avulsion fracture of the spinous process of the lower cervical vertebrae. This oblique fracture of the base of the spinous process, classically C7, derived its name in the 1930s when Australian miners lifted a heavy shovelful of clay causing an abrupt flexion of the head, in opposition to the stabilizing force of the strong supraspinous muscle, resulting in an avulsion fracture of the spinous process. An extension teardrop fracture involves a hyperextension injury in which the anterior longitudinal ligament avulses the inferior portion of the anterior vertebral body at its insertion. The second cervical vertebra is the most common location for an extension teardrop fracture

Subdural hematomas are most likely characterized by which of the following findings? (A) may result from a skull fracture across the middle meningeal artery (B) formed deep within the brain tissue and usually caused by shearing or tensile forces that mechanically stretch and tear deep small-caliber arterioles (C) are associated with traumatic blood within the cerebrospinal fluid (CSF) and meningeal intima caused by small tears of subarachnoid vessels (D) a collection of clear, xanthochromic blood-tinged fluid in the dural space (E) a rupture of superficial bridging vessels with rapid movement of the head, as in acceleration-deceleration injuries

60. (E) Subdural hematomas are usually the result of venous bleeding, causing blood clots to form between the dura and the brain. The most common mechanisms are acceleration-deceleration injuries, which usually cause the superficial bridging vessels to rupture. Subarachnoid hemorrhage caused by trauma is secondary to small tears of the subarachnoid vessels. Blood will collect within the CSF and meningeal intima. A subdural hygroma is formed by clear xanthochromic dural space fluid. Although the pathogenesis is unknown, it is thought to be from a tear in the arachnoid space that permits CSF to escape into the dural space. Intracerebral hematomas are the result of mechanical stretching and tearing of deep small-caliber arterioles of the brain. These mechanical forces usually are the result of the brain being propelled against irregular surfaces in the cranial vault. Most of the intracerebral hematomas are in the frontal and temporal lobes.

Maxillofacial trauma in a 35-year-old woman is most likely associated with (A) falls (B) motor vehicle collision (MVC) (C) work related (D) sports related (E) domestic violence

65. (E) The source of facial injuries varies by city or rural settings. There is a high incidence of domestic violence associated with a female patient presenting with an orbital fracture. Statistics reports that about 25% of facial injuries seen in the ED are associated with domestic violence. Facial injuries associated with falls are seen more frequently with the elderly and pediatric populations. Rural hospitals see higher rates of facial trauma associated with MVC and sports-related injuries.

Which of the following diagnoses is associated with a non-anion gap acidosis? (A) renal tubular acidosis (B) methanol poisoning (C) ketoacidosis (D) uremia associated with renal failure (E) ethylene glycol poisoning

9. (A) Renal tubular acidosis is an example of a normal anion gap acidosis in which there is an increase in chloride and a loss of bicarbonate. The other choices of diagnoses are associated with a high anion gap metabolic acidosis 643 or a metabolic alkalosis. (Morris, 2008, p. 831)

A 6-year-old child (20 kg) presents to the ED after being struck by a car. The child suffered chest, head, and abdominal trauma. The vital signs demonstrate a blood pressure of 80/40 mm Hg, a pulse of 170/min, and a respiratory rate of 40/min. After the airway is managed, which is the most appropriate initial fluid therapy? (A) dextrose 5% water at 100 mL/h (B) normal saline 5 mL/kg bolus, then 100 mL/h (C) lactated Ringer's 20 mL/kg bolus (D) lactated Ringer's at 20 mL/h (E) normal saline 200 mL bolus

C) A crystalloid fluid bolus of 20 mg/kg is recommended for initial resuscitation in hypovolemic pediatric trauma patients. If 40 mg/kg of crystalloid fluids does not lead to improvement of the hypovolemic state, administration of packed red blood cells should be initiated

A 58-year-old man presents to the ED hypothermic after an environmental exposure to cold weather and snow. The patient's core temperature is 85.5°F. Which of the following is the most accurate statement regarding this scenario? (A) Shivering is common. (B) An Osborne (J) wave is pathognomic for hypothermia. (C) Rough handling can produce serious dysrhythmias. (D) A nasogastric tube should be inserted to protect the airway from regurgitation. (E) The patient is in an excitation phase of hypothermia.

C) Mild hypothermia is defined as a temperature from 32°C to 35°C (89.6°F- 95°F). In mild hypothermia, the body responds by increasing metabolic activity to produce heat. This is known as the excitation or the responsive phase. When the temperature drops to less than 32°C (89.6°F), bodily functions slow down, giving way to the adynamic phase. As metabolism slows, there is a decrease in both oxygen utilization and carbon dioxide production. As the body temperature falls to less than 30°C to 32°C (86°F-89.6°F), shivering will cease. Hypothermia may induce life-threatening dysrhythmias and ECG changes. A characteristic, but not pathognomonic, ECG finding in hypothermia is the Osborne (J) wave. This abnormal wave is a slow, positive deflection at the end of the QRS complex.

A 28-year-old pregnant woman in her third trimester presents with a blood pressure (BP) of 164/98 mm Hg. Which of the following medications is the agent of choice in this scenario? (A) nitroglycerin (B) magnesium (C) hydralazine (D) captopril (E) hydrochlorothiazide

C) Pregnancy-induced hypertension (PIH) is defined as a blood pressure (BP) reading of 140/90 mm Hg or higher. The subtypes of PIH include hypertension without proteinuria or edema, preeclampsia (hypertension with proteinuria or edema), and eclampsia (seizures in the pregnant patient with signs of preeclampsia). PIH occurs in about 5% of pregnancies and eclampsia occurs in less than 1 in 2,000 deliveries. Hydralazine is the most common antihypertensive used in PIH. The typical dose is 5 mg IV and repeated in a 644 dose of 5 to 10 mg IV every 20 minutes as needed to keep the diastolic BP less than 110 mm Hg. Other agents used with some degree of success include nifedipine, nitroprusside, and labetalol. Magnesium sulfate is used primarily in pregnancy to terminate ongoing seizures and prevent further seizures. This is usually accomplished by keeping the serum magnesium levels at 4 to 7 mg/dL. In general, nitroglycerin and hydrochlorothiazide are not indicated in PIH. (Newton and Calder, 2006, pp. 2762-2765)

A 71-year-old woman presents to the ED with malaise, acute headache, and a rapidly decreasing visual acuity. Which of the following best describes this condition? (A) glaucoma (B) multiple sclerosis (C) temporal arteritis (D) myasthenia gravis (E) viral encephalitis

C) Temporal arteritis is a vasculitis usually occurring in the elderly involving the temporal and external carotid artery. Left untreated, temporal arteritis may result in bilateral blindness. Patients typically present with unilateral, excruciating, burning pain over the affected artery. The disease is often associated with polymyalgia rheumatica and may present with systemic involvement including fever, polymyalgia, malaise, weight loss, and anorexia. Patients complain of decreased visual acuity, and the examination reveals a tender, inflamed temporal artery. Acute narrow angle glaucoma is characterized by a sudden onset of severe pain localized to the affected eye with halos around lights, blurriness, scotomas, and sometimes nausea and vomiting. Typical physical examination findings reveal a red eye with fixed, mid-dilated pupil, corneal clouding, and a shallow anterior chamber. Multiple sclerosis is a demyelinating disease that affects the CNS and is characterized by recurrent attacks of focal and multifocal neurologic deficits. Multiple sclerosis may manifest as an optic neuritis, which is an inflammatory condition of the optic nerve. Optic neuritis typically manifests as eye pain and visual impairment. Myasthenia gravis is characterized by episodic muscle weakness caused by loss or dysfunction of acetylcholine receptors. Common symptoms related to ocular muscle involvement include ptosis, diplopia, and muscle fatigability after exercise. Viral encephalitis is an acute inflammatory disease of the brain caused by direct viral invasion (eg, arbovirus, poliovirus, echovirus, and coxsackie virus). Common symptoms include fever, malaise, headache, vomiting, stiff neck, seizures, and cranial nerve abnormalities.


Conjuntos de estudio relacionados

(1.1b) Ancient Indian Scientists Part 1- Astronomers and Mathematicians

View Set

Nursing Care of Adults -> Exam #3

View Set

Chapter 28 Face and Neck Injuries:

View Set